LSAT and Law School Admissions Forum

Get expert LSAT preparation and law school admissions advice from PowerScore Test Preparation.

 Administrator
PowerScore Staff
  • PowerScore Staff
  • Posts: 8927
  • Joined: Feb 02, 2011
|
#23483
Complete Question Explanation

Strengthen—PR. The correct answer choice is (A)

This question type is rare, so it is important that we have a precise understanding of what is meant by the language in the question stem: "The committee member's argument conforms most closely to which one of the following principles?" Note that the answer choices are in the form of "should" statements. Our task is to find the answer choice that most accurately summarizes the thrust of the committee member's argument, what the committee member "is driving at." The correct answer will be a principle that follows the line of reasoning used without being to general or specific and without ignoring certain key "moving parts" within the stimulus. If we do not see this at the beginning, we could take a quick look at the answer choices and realize that each answer choice is a generalized statement that has the potential to apply here. The four wrong answers will somehow significantly deviate from what the committee member is "driving at." This is something like an abstract Main Point question. In order to test the answer choices against the stimulus, we must know exactly what the stimulus is saying.

If our reading is precise, we will notice that the actual conclusion involves a recommendation regarding voting. The committee will vote on the person to be the head of the military, and the committee member is arguing that anyone with a history of excessive drinking should not be voted into the position. Two statements are used to support this conclusion: that persons who drink excessively would not be able to command certain subunits of the military and that "leadership must be established from the top down." The difference between these statements is crucial: the former statement is purely factual (such a person would be barred) and the latter statement is a value judgment (leadership should be a certain way). The committee member could have spelled out his or her thinking more completely, but we can see that the argument relies critically upon an assertion that the military's top leader SHOULD have certain characteristics and one of these is the absence of excessive drinking because a leader who drank excessively would be giving orders to people in jobs that the leader would not have been permitted to perform. We must consider each answer choice to see just how close each one mirrors this assertion.

Answer choice (A): This is the correct answer choice. Here, we do not have a mention of excessive drinking, but we do not need one. This answer choice is more general than the argument in the stimulus because it covers all organization types and it covers all reasons for being prohibited from a certain job. Clearly, this answer choice would address people who drink excessively because those people "would be barred from important jobs" [controlling missiles, bombers, and fighters could reasonably be thought of as important tasks]. People who drink excessively would not be able to lead the military because they would be barred from important subordinate jobs. We needed a principle that is closest to the principle underlying the committee members statement, so we must look at the other answer choices. However, we should feel confident in our analysis so far.

Answer choice (B): This answer choice should be quickly discarded because the committee member does not discuss whether a job candidate has served in a certain capacity, only whether a job candidate COULD do so. Furthermore, we can imagine that the committee member would not assert that a job candidate must have been a missile wing commander and a bomber squadron leader and a fighter contingent leader. The military is a large organization and such a requirement would take several lifetimes. This answer choice has no chance of matching up with the committee member's assertion.

Answer choice (C): This is a tricky answer choice, but it provides a great learning opportunity. If we really understood the stimulus, we should be pretty comfortable with answer choice A, which is quite different from this one. Answer choice A discusses prohibitions and answer choice C discusses affirmative requirements. If true, answer choice C would require that an army general from infantry know how to drive a submarine or fly a jet before getting the top job. No modern military operates this way, and no organization of sufficient size could operate this way. Once we realize this, we can confidently remove this answer choice from consideration.

Answer choice (D): This might be the most frequently selected wrong answer because test-takers lose their bearings. Remember that we are picking the one answer choice that is closest to the original argument. With this answer choice, we have a statement that WOULD SUPPORT the committee member's final conclusion that the committee should not vote for a person who drinks excessively, but the statement significantly deviates from the LINE OF REASONING we are comparing. The correct answer for this type of problem will contain the structure found in the stimulus, including all the "elements" of the argument. This answer choice directly references excessive drinking, but leapfrogs the entire premise regarding qualification for subordinate positions of leadership. For this reason, the two principles' moving parts do not match up, and we can discard this answer choice.

Answer choice (E): Much like answer choice C, this answer choice tries to get into the details of being qualified for certain jobs within the military. As we have noted, this is not what the committee member is driving at. It would be very difficult to imagine knowledgeable military figures arguing that the top commander should be required to know how to run a missile command, which probably involves very specialized training over several years. Commanding an entire military is hard enough without having the added burden of learning "rocket science."
 mattm
  • Posts: 50
  • Joined: Jun 10, 2014
|
#15487
Hello,

I was wondering why answer choice A is correct .....particularly the wording of "important jobs ". In the answer choice

By the wording of the stimulus it doesn't say the relative importance of the three jobs ...( although if these jobs weren't important it's likely that the author wouldn't use them as support for his conclusion )

From a broad sense ( and in this question )....when Is it ok to make an assumption like that the jobs are important ?......I could see something like this being a trap answer choice in another question, The better I've gotten at the LSAT I've noticed the trends in the wrong answer choices and that the LSAT tests unwarranted assumptions and unwarranted inferences so I thought it was one of those cases .

Thanks !
 David Boyle
PowerScore Staff
  • PowerScore Staff
  • Posts: 836
  • Joined: Jun 07, 2013
|
#15492
mattm wrote:Hello,

I was wondering why answer choice A is correct .....particularly the wording of "important jobs ". In the answer choice

By the wording of the stimulus it doesn't say the relative importance of the three jobs ...( although if these jobs weren't important it's likely that the author wouldn't use them as support for his conclusion )

From a broad sense ( and in this question )....when Is it ok to make an assumption like that the jobs are important ?......I could see something like this being a trap answer choice in another question, The better I've gotten at the LSAT I've noticed the trends in the wrong answer choices and that the LSAT tests unwarranted assumptions and unwarranted inferences so I thought it was one of those cases .

Thanks !
Hello mattm,

First, I believe it's question 13, by the way.

Answer A is correct since the stimulus can be razored down to, "Since the guy can't even run a missile wing or some other jobs, how can he run the whole military?" and answer A, "No one who would be barred from important jobs in an organization should lead that organization.", encapsulates that nicely.

The stimulus may not use the word "important" to describe the three jobs, but by implication, especially since the issue is of someone being a *commander* of a missile wing or whatever, the job is obviously important. Just by common sense, I suppose. (Unwarranted assumptions are bad, but if you cannot make even common-sense assumptions, that may be even worse!)

Hope that helps,
David
 JaneBlueSky
  • Posts: 10
  • Joined: Aug 14, 2016
|
#32102
Hey, I read through the explanation but still not clear about why the answer D is wrong. What the Committee is saying matches both A and D.

Could some one provide a more insightful understanding of the difference between A and D?

Thanks.

Jane
 David Boyle
PowerScore Staff
  • PowerScore Staff
  • Posts: 836
  • Joined: Jun 07, 2013
|
#32123
JaneBlueSky wrote:Hey, I read through the explanation but still not clear about why the answer D is wrong. What the Committee is saying matches both A and D.

Could some one provide a more insightful understanding of the difference between A and D?

Thanks.

Jane

Hello Jane,

The stimulus says, "an individual whose history of excessive drinking is such that that person would be barred...", leaving it open that someone could drink excessively--maybe once a year?--in a way that would not get him or her barred from command. So, answer D, "No one who drinks excessively should hold a leadership position anywhere along the military’s chain of command", is not provably correct.
Nor does answer D have the nice generality that answer A, "No one who would be barred from important jobs in an organization should lead that organization", has. Answer A also chimes with the generality of "Leadership must be established from the top down."

Hope this helps,
David
 lathlee
  • Posts: 652
  • Joined: Apr 01, 2016
|
#42633
Hi. I see a strong presence of Conditional relationships in Question stem and in answer choices as well. Can you guys possibly explain the correct answer choice with the explanation of Conditional relationships?
 Adam Tyson
PowerScore Staff
  • PowerScore Staff
  • Posts: 5153
  • Joined: Apr 14, 2011
|
#42724
We could, lathlee! If I were to do this conditionally, with a diagram, I would probably diagram the stimulus as:

History of drinking bars someone from some jobs :arrow: lead the organization that includes those jobs

As you can see, that is essentially answer choice A. Winner!

Side note: I would not diagram the last sentence, as it seems to be nonessential. If I did, though, it would be:

Leader :arrow: Top Down
 lathlee
  • Posts: 652
  • Joined: Apr 01, 2016
|
#44667
Hi. I still cannot comprehend the question type as the Strengthen-PR, i keep seeing it as of Parallel PR Question. how does this question differ from other Question 1st type family? Also, I fail to see any answer choices here , helps to strength the conclusion of the question stem
 Malila Robinson
PowerScore Staff
  • PowerScore Staff
  • Posts: 296
  • Joined: Feb 01, 2018
|
#47143
Hi lathlee,
First let's address your question of how this is a Strengthen question. The Administrator (above) wrote out a detailed explanation, which I will copy here and then expand upon.

"This question type is rare, so it is important that we have a precise understanding of what is meant by the language in the question stem: "The committee member's argument conforms most closely to which one of the following principles?" Note that the answer choices are in the form of "should" statements. Our task is to find the answer choice that most accurately summarizes the thrust of the committee member's argument, what the committee member "is driving at." The correct answer will be a principle that follows the line of reasoning used without being to general or specific and without ignoring certain key "moving parts" within the stimulus. If we do not see this at the beginning, we could take a quick look at the answer choices and realize that each answer choice is a generalized statement that has the potential to apply here. The four wrong answers will somehow significantly deviate from what the committee member is "driving at." This is something like an abstract Main Point question. In order to test the answer choices against the stimulus, we must know exactly what the stimulus is saying."

So, the reason this is a 2nd Family Question instead of a 1st Family Question is that the question stem is asking you to choose a principle that, if true, would provide a basis for the committee member's argument. A 1st Family Question would have made it more clear that because the committee member's argument is true, one of the principles in the answer choices is true. So it may help to think of what the question is emphasizing. Is the committee member's argument matching the principal? If so that is a 2nd family because the truth is in the answers. Or is the principal matching the committee member's argument? If so that would be a 1st family because the truth would be in the stimulus.

As for why Answer A strengthens the argument, again the Administrator has explained this, so I'll copy it here and then try to add a bit more:
"Answer choice (A): This is the correct answer choice. Here, we do not have a mention of excessive drinking, but we do not need one. This answer choice is more general than the argument in the stimulus because it covers all organization types and it covers all reasons for being prohibited from a certain job. Clearly, this answer choice would address people who drink excessively because those people "would be barred from important jobs" [controlling missiles, bombers, and fighters could reasonably be thought of as important tasks]. People who drink excessively would not be able to lead the military because they would be barred from important subordinate jobs. We needed a principle that is closest to the principle underlying the committee members statement, so we must look at the other answer choices. However, we should feel confident in our analysis so far."

Since this is a Strengthen question we do not have to match the argument exactly, we just need to provide a principle that, if true, would explain where the committee member's argument is coming from. A principle is a broad rule that can apply to different situations. In this case the principle in A provides a basis for the committee member's argument, so if it is true, it would strengthen the committee member's position.
Hope that helps,
-Malila
 Blueballoon5%
  • Posts: 156
  • Joined: Jul 13, 2015
|
#47855
Could you help me understand the meaning of answer choice E? The double negative confuses me. What is this sentence trying to say?

Get the most out of your LSAT Prep Plus subscription.

Analyze and track your performance with our Testing and Analytics Package.